Each sequence that is unbounded above / below has a subsequence that converges (improperly) to $+ infty /...












0















To Prove: Each sequence that is unbounded above / below has a subsequence
that converges (improperly) to $+ infty / −infty$. Conclude that each sequence
of real numbers has a properly or improperly convergent subsequence.




Whenever a sequence is unbounded, we have that it has an increasing/decreasing subsequence. Why is this the case? For any number $M$ you give me, there exists an $min mathbb{N}$ such that $a_m$>M. We choose the subsequence ${a_i, a_j, a_k dots }$ such that the index set is strictly increasing/decreasing and also the terms are strictly increasing/decreasing. Does this argument work? how can I formalise this better?



An example of my method applied would be the following sequence: ${0,1,2,1,0,3,4,5,4,3,6 dots dots }$ My method would pick the following subsequence ${0, 1,2,3,4,5,6 dots}$. This is the sequence $a_l=l$ where $l$ is now a new index.



Whenever a sequence is bounded it has a convergent subsequence by Bolzano Weierstrass (it has an accumulation point, hence the subsequence converges to this accumulation point.)










share|cite|improve this question


















  • 1




    Simply build recursively a subsequence $(a_{n_j})$ such that $a_{n_0}=a_0$, say, and, for every $j$, $$a_{n_{j+1}}geqslant a_{n_j}+1$$
    – Did
    Nov 25 at 6:15












  • That's a nice way to put it.
    – Wesley Strik
    Nov 25 at 6:18
















0















To Prove: Each sequence that is unbounded above / below has a subsequence
that converges (improperly) to $+ infty / −infty$. Conclude that each sequence
of real numbers has a properly or improperly convergent subsequence.




Whenever a sequence is unbounded, we have that it has an increasing/decreasing subsequence. Why is this the case? For any number $M$ you give me, there exists an $min mathbb{N}$ such that $a_m$>M. We choose the subsequence ${a_i, a_j, a_k dots }$ such that the index set is strictly increasing/decreasing and also the terms are strictly increasing/decreasing. Does this argument work? how can I formalise this better?



An example of my method applied would be the following sequence: ${0,1,2,1,0,3,4,5,4,3,6 dots dots }$ My method would pick the following subsequence ${0, 1,2,3,4,5,6 dots}$. This is the sequence $a_l=l$ where $l$ is now a new index.



Whenever a sequence is bounded it has a convergent subsequence by Bolzano Weierstrass (it has an accumulation point, hence the subsequence converges to this accumulation point.)










share|cite|improve this question


















  • 1




    Simply build recursively a subsequence $(a_{n_j})$ such that $a_{n_0}=a_0$, say, and, for every $j$, $$a_{n_{j+1}}geqslant a_{n_j}+1$$
    – Did
    Nov 25 at 6:15












  • That's a nice way to put it.
    – Wesley Strik
    Nov 25 at 6:18














0












0








0








To Prove: Each sequence that is unbounded above / below has a subsequence
that converges (improperly) to $+ infty / −infty$. Conclude that each sequence
of real numbers has a properly or improperly convergent subsequence.




Whenever a sequence is unbounded, we have that it has an increasing/decreasing subsequence. Why is this the case? For any number $M$ you give me, there exists an $min mathbb{N}$ such that $a_m$>M. We choose the subsequence ${a_i, a_j, a_k dots }$ such that the index set is strictly increasing/decreasing and also the terms are strictly increasing/decreasing. Does this argument work? how can I formalise this better?



An example of my method applied would be the following sequence: ${0,1,2,1,0,3,4,5,4,3,6 dots dots }$ My method would pick the following subsequence ${0, 1,2,3,4,5,6 dots}$. This is the sequence $a_l=l$ where $l$ is now a new index.



Whenever a sequence is bounded it has a convergent subsequence by Bolzano Weierstrass (it has an accumulation point, hence the subsequence converges to this accumulation point.)










share|cite|improve this question














To Prove: Each sequence that is unbounded above / below has a subsequence
that converges (improperly) to $+ infty / −infty$. Conclude that each sequence
of real numbers has a properly or improperly convergent subsequence.




Whenever a sequence is unbounded, we have that it has an increasing/decreasing subsequence. Why is this the case? For any number $M$ you give me, there exists an $min mathbb{N}$ such that $a_m$>M. We choose the subsequence ${a_i, a_j, a_k dots }$ such that the index set is strictly increasing/decreasing and also the terms are strictly increasing/decreasing. Does this argument work? how can I formalise this better?



An example of my method applied would be the following sequence: ${0,1,2,1,0,3,4,5,4,3,6 dots dots }$ My method would pick the following subsequence ${0, 1,2,3,4,5,6 dots}$. This is the sequence $a_l=l$ where $l$ is now a new index.



Whenever a sequence is bounded it has a convergent subsequence by Bolzano Weierstrass (it has an accumulation point, hence the subsequence converges to this accumulation point.)







real-analysis sequences-and-series






share|cite|improve this question













share|cite|improve this question











share|cite|improve this question




share|cite|improve this question










asked Nov 25 at 6:06









Wesley Strik

1,534422




1,534422








  • 1




    Simply build recursively a subsequence $(a_{n_j})$ such that $a_{n_0}=a_0$, say, and, for every $j$, $$a_{n_{j+1}}geqslant a_{n_j}+1$$
    – Did
    Nov 25 at 6:15












  • That's a nice way to put it.
    – Wesley Strik
    Nov 25 at 6:18














  • 1




    Simply build recursively a subsequence $(a_{n_j})$ such that $a_{n_0}=a_0$, say, and, for every $j$, $$a_{n_{j+1}}geqslant a_{n_j}+1$$
    – Did
    Nov 25 at 6:15












  • That's a nice way to put it.
    – Wesley Strik
    Nov 25 at 6:18








1




1




Simply build recursively a subsequence $(a_{n_j})$ such that $a_{n_0}=a_0$, say, and, for every $j$, $$a_{n_{j+1}}geqslant a_{n_j}+1$$
– Did
Nov 25 at 6:15






Simply build recursively a subsequence $(a_{n_j})$ such that $a_{n_0}=a_0$, say, and, for every $j$, $$a_{n_{j+1}}geqslant a_{n_j}+1$$
– Did
Nov 25 at 6:15














That's a nice way to put it.
– Wesley Strik
Nov 25 at 6:18




That's a nice way to put it.
– Wesley Strik
Nov 25 at 6:18










1 Answer
1






active

oldest

votes


















0














We build a recursive subsequence defined by:
$$a_{n_0}=a_0 land n_{j+1} > n_j land a_{n_{j+1}} geq a_{n_j}+1 $$
We can do so because the set is unbounded, so we can always handpick the next term that is greater than our previous one, because there should never exist a bound. Suppose we would not be able to pick a greater term, we would get a contradiction since $a_{n_j}+1$ would be a bound for the entire sequence, which was supposed to be unbounded.



We also notice that the index set is strictly increasing and that the subsequence itself is increasing, the question is, is it increasing fast enough?



Notice that we take some arbitrary term for the sequence and handpick the next one that is at least greater than the current one plus $1$. This looks very familiar:
$$ a_{n+1} = a_{n} + 1$$
Is equivalent to linear growth, or also in a direct formula:
$$ a_n = n + a_0$$
We have thus desired that our sequence grows at least as fast as this sequence. However, we know that this sequence diverges. Because or minorant diverges, our original subsequence must also diverge. $square$






share|cite|improve this answer





















    Your Answer





    StackExchange.ifUsing("editor", function () {
    return StackExchange.using("mathjaxEditing", function () {
    StackExchange.MarkdownEditor.creationCallbacks.add(function (editor, postfix) {
    StackExchange.mathjaxEditing.prepareWmdForMathJax(editor, postfix, [["$", "$"], ["\\(","\\)"]]);
    });
    });
    }, "mathjax-editing");

    StackExchange.ready(function() {
    var channelOptions = {
    tags: "".split(" "),
    id: "69"
    };
    initTagRenderer("".split(" "), "".split(" "), channelOptions);

    StackExchange.using("externalEditor", function() {
    // Have to fire editor after snippets, if snippets enabled
    if (StackExchange.settings.snippets.snippetsEnabled) {
    StackExchange.using("snippets", function() {
    createEditor();
    });
    }
    else {
    createEditor();
    }
    });

    function createEditor() {
    StackExchange.prepareEditor({
    heartbeatType: 'answer',
    autoActivateHeartbeat: false,
    convertImagesToLinks: true,
    noModals: true,
    showLowRepImageUploadWarning: true,
    reputationToPostImages: 10,
    bindNavPrevention: true,
    postfix: "",
    imageUploader: {
    brandingHtml: "Powered by u003ca class="icon-imgur-white" href="https://imgur.com/"u003eu003c/au003e",
    contentPolicyHtml: "User contributions licensed under u003ca href="https://creativecommons.org/licenses/by-sa/3.0/"u003ecc by-sa 3.0 with attribution requiredu003c/au003e u003ca href="https://stackoverflow.com/legal/content-policy"u003e(content policy)u003c/au003e",
    allowUrls: true
    },
    noCode: true, onDemand: true,
    discardSelector: ".discard-answer"
    ,immediatelyShowMarkdownHelp:true
    });


    }
    });














    draft saved

    draft discarded


















    StackExchange.ready(
    function () {
    StackExchange.openid.initPostLogin('.new-post-login', 'https%3a%2f%2fmath.stackexchange.com%2fquestions%2f3012485%2feach-sequence-that-is-unbounded-above-below-has-a-subsequence-that-converges%23new-answer', 'question_page');
    }
    );

    Post as a guest















    Required, but never shown

























    1 Answer
    1






    active

    oldest

    votes








    1 Answer
    1






    active

    oldest

    votes









    active

    oldest

    votes






    active

    oldest

    votes









    0














    We build a recursive subsequence defined by:
    $$a_{n_0}=a_0 land n_{j+1} > n_j land a_{n_{j+1}} geq a_{n_j}+1 $$
    We can do so because the set is unbounded, so we can always handpick the next term that is greater than our previous one, because there should never exist a bound. Suppose we would not be able to pick a greater term, we would get a contradiction since $a_{n_j}+1$ would be a bound for the entire sequence, which was supposed to be unbounded.



    We also notice that the index set is strictly increasing and that the subsequence itself is increasing, the question is, is it increasing fast enough?



    Notice that we take some arbitrary term for the sequence and handpick the next one that is at least greater than the current one plus $1$. This looks very familiar:
    $$ a_{n+1} = a_{n} + 1$$
    Is equivalent to linear growth, or also in a direct formula:
    $$ a_n = n + a_0$$
    We have thus desired that our sequence grows at least as fast as this sequence. However, we know that this sequence diverges. Because or minorant diverges, our original subsequence must also diverge. $square$






    share|cite|improve this answer


























      0














      We build a recursive subsequence defined by:
      $$a_{n_0}=a_0 land n_{j+1} > n_j land a_{n_{j+1}} geq a_{n_j}+1 $$
      We can do so because the set is unbounded, so we can always handpick the next term that is greater than our previous one, because there should never exist a bound. Suppose we would not be able to pick a greater term, we would get a contradiction since $a_{n_j}+1$ would be a bound for the entire sequence, which was supposed to be unbounded.



      We also notice that the index set is strictly increasing and that the subsequence itself is increasing, the question is, is it increasing fast enough?



      Notice that we take some arbitrary term for the sequence and handpick the next one that is at least greater than the current one plus $1$. This looks very familiar:
      $$ a_{n+1} = a_{n} + 1$$
      Is equivalent to linear growth, or also in a direct formula:
      $$ a_n = n + a_0$$
      We have thus desired that our sequence grows at least as fast as this sequence. However, we know that this sequence diverges. Because or minorant diverges, our original subsequence must also diverge. $square$






      share|cite|improve this answer
























        0












        0








        0






        We build a recursive subsequence defined by:
        $$a_{n_0}=a_0 land n_{j+1} > n_j land a_{n_{j+1}} geq a_{n_j}+1 $$
        We can do so because the set is unbounded, so we can always handpick the next term that is greater than our previous one, because there should never exist a bound. Suppose we would not be able to pick a greater term, we would get a contradiction since $a_{n_j}+1$ would be a bound for the entire sequence, which was supposed to be unbounded.



        We also notice that the index set is strictly increasing and that the subsequence itself is increasing, the question is, is it increasing fast enough?



        Notice that we take some arbitrary term for the sequence and handpick the next one that is at least greater than the current one plus $1$. This looks very familiar:
        $$ a_{n+1} = a_{n} + 1$$
        Is equivalent to linear growth, or also in a direct formula:
        $$ a_n = n + a_0$$
        We have thus desired that our sequence grows at least as fast as this sequence. However, we know that this sequence diverges. Because or minorant diverges, our original subsequence must also diverge. $square$






        share|cite|improve this answer












        We build a recursive subsequence defined by:
        $$a_{n_0}=a_0 land n_{j+1} > n_j land a_{n_{j+1}} geq a_{n_j}+1 $$
        We can do so because the set is unbounded, so we can always handpick the next term that is greater than our previous one, because there should never exist a bound. Suppose we would not be able to pick a greater term, we would get a contradiction since $a_{n_j}+1$ would be a bound for the entire sequence, which was supposed to be unbounded.



        We also notice that the index set is strictly increasing and that the subsequence itself is increasing, the question is, is it increasing fast enough?



        Notice that we take some arbitrary term for the sequence and handpick the next one that is at least greater than the current one plus $1$. This looks very familiar:
        $$ a_{n+1} = a_{n} + 1$$
        Is equivalent to linear growth, or also in a direct formula:
        $$ a_n = n + a_0$$
        We have thus desired that our sequence grows at least as fast as this sequence. However, we know that this sequence diverges. Because or minorant diverges, our original subsequence must also diverge. $square$







        share|cite|improve this answer












        share|cite|improve this answer



        share|cite|improve this answer










        answered Nov 27 at 0:21









        Wesley Strik

        1,534422




        1,534422






























            draft saved

            draft discarded




















































            Thanks for contributing an answer to Mathematics Stack Exchange!


            • Please be sure to answer the question. Provide details and share your research!

            But avoid



            • Asking for help, clarification, or responding to other answers.

            • Making statements based on opinion; back them up with references or personal experience.


            Use MathJax to format equations. MathJax reference.


            To learn more, see our tips on writing great answers.





            Some of your past answers have not been well-received, and you're in danger of being blocked from answering.


            Please pay close attention to the following guidance:


            • Please be sure to answer the question. Provide details and share your research!

            But avoid



            • Asking for help, clarification, or responding to other answers.

            • Making statements based on opinion; back them up with references or personal experience.


            To learn more, see our tips on writing great answers.




            draft saved


            draft discarded














            StackExchange.ready(
            function () {
            StackExchange.openid.initPostLogin('.new-post-login', 'https%3a%2f%2fmath.stackexchange.com%2fquestions%2f3012485%2feach-sequence-that-is-unbounded-above-below-has-a-subsequence-that-converges%23new-answer', 'question_page');
            }
            );

            Post as a guest















            Required, but never shown





















































            Required, but never shown














            Required, but never shown












            Required, but never shown







            Required, but never shown

































            Required, but never shown














            Required, but never shown












            Required, but never shown







            Required, but never shown







            Popular posts from this blog

            Quarter-circle Tiles

            build a pushdown automaton that recognizes the reverse language of a given pushdown automaton?

            Mont Emei